How is D wrong?
Makes no sense
Tyler808 on July 27, 2022
  • June 2004 LSAT
  • SEC3
  • Q1
1
Reply
Thinking out loud...Is this the correct interpr...
Hi there, I would imagine I'm among the majority of people who got the answer incorrect. I sel...
JonJay on April 27, 2022
  • June 2004 LSAT
  • SEC3
  • Q25
1
Reply
chose D over A because of “often” in question p...
Like many others, I also chose D over A because I thought that the word “often” that was stated i...
meghelle on March 25, 2022
  • June 2004 LSAT
  • SEC3
  • Q25
1
Reply
makes no sense
some FCA are MN all OC are FCA you can have an instance where FCA-some-OC and FCA-some-MN b...
medasmx@protonmail.com on January 22, 2022
  • June 2004 LSAT
  • SEC3
  • Q16
1
Reply
Question
Could you please explain why answer choice D is incorrect?
erojas on July 27, 2021
  • June 2004 LSAT
  • SEC3
  • Q25
8
Replies
Can this be better explained than just no answer?
This is a lot of complicated wording and premises, it would be helpful to have an explanation on ...
andreaskormusis on January 21, 2021
  • June 2004 LSAT
  • SEC3
  • Q12
3
Replies
Principle Questions
It seems that if it is a principle question, then the correct Answer choice must be supported bas...
zgnewquist on January 13, 2021
  • June 2004 LSAT
  • SEC3
  • Q11
3
Replies
Why not C?
I get why A is correct but why would C be wrong?
liwenong28 on November 11, 2020
  • June 2004 LSAT
  • SEC3
  • Q1
1
Reply
Why is E wrong? Thanks
Why is E wrong? Thanks
jingjingxiao11111@gmail.com on November 10, 2020
  • June 2004 LSAT
  • SEC3
  • Q19
1
Reply
Why is E incorrect?
Why is E incorrect?
Shiyi-Zhang on October 26, 2020
  • June 2004 LSAT
  • SEC3
  • Q22
2
Replies
Can you please explain this answer?
Thank you in advance.
amvrba1 on April 16, 2020
  • June 2004 LSAT
  • SEC3
  • Q10
2
Replies
Clarification
Effective Teacher-Most-ECC NotECC-Some-Effective Teacher Effective Teacher-GC Combine st...
gharibiannick on April 12, 2020
  • June 2004 LSAT
  • SEC3
  • Q9
1
Reply
Please explain.
Could you please explain why (B) is correct and why (C) is incorrect? Thank you.
JShahar on January 14, 2020
  • June 2004 LSAT
  • SEC3
  • Q21
1
Reply
Explanation
Can you please explain this question? I had the answer choices down to B, D, and E but I would li...
sydbrown6 on February 4, 2016
  • June 2004 LSAT
  • SEC3
  • Q24
1
Reply
Need your help
Could you please show me how draw diagram of the reasoning method on the stimulus? I don't think ...
Batman on August 25, 2014
  • June 2004 LSAT
  • SEC3
  • Q25
2
Replies